PT1.S3.Q4 - I need help understanding why A is wrong

cbookercbooker Live Member
edited July 2023 in Logical Reasoning 28 karma

I need help understanding why A is wrong. I kinda get why D is right even though it says "unsurpassed", which seems pretty excessive to me.

Admin Note: Edited title. Please use the format: "PT#.S#.Q# - brief description of the question."

Comments

  • KangtimeKangtime Alum Member
    edited July 2023 74 karma

    These older tests in the single digit #s compared to more modern PTs are so much harder in my opinion! I'll try to explain:

    The reason why A is wrong is because of the phrase only if. if it was replaced with if, then it would be the correct answer.

    only if is a necessary condition indicator. But what we can take from our stimulus is that military deterrence is one way of achieving deterrence from a would-be aggressor from attacking (a sufficient condition!). We can easily imagine a situation where a country will go seriously bankrupt if it started a war, and therefore decided not to. Or too much loss in their population. Or a fear of a civil resistance. Or not enough soldiers to fight. The point being, there can be many other reasons why a country would not start a war, and all of that would be compatible with the information given in the stimulus. (this is also why C is wrong - just because a country didn't attack doesn't mean it was due to military deterrence). Therefore, military deterrence is NOT a necessary condition for deterrence from international conflict. That's why A is wrong. (In this respect, the answer choice would also be correct if it started with "military deterrence can effectively deter a would -be aggressor nation from attacking only if...", because then the answer choice qualifies HOW the country is being deterred; if it doesn't have this qualification as the real answer choice A does not, then many other possible reasons can independently deter a country from attacking even without military deterrence.)

    The word 'unsurpassed' is put in answer D to make sure that the country under question is absolutely #1 when it comes to military power. Because if, let's say, it was #10, then it actually might not be in the country's best interest to let their military power known to countries with military power stronger than itself. For example, If country that's #3 in military power is a potential aggressor nation to the country in question, which is #10 in military power, and #3 country finds out that the other country has a weaker military power than them, then #3 country could actually be even more inclined to start a war now. The usage of the word 'unsurpassed' gets rid of this possibility.

    So if a country wants to deter war, and it has the strongest military power on the planet, then we could infer that it would want other countries to know about its military capabilities - because the stimulus tells us (and therefore we take it as absolute truth) that military deterrence works.

    Maybe it's just me, but I really find these older questions to be challenging because of how they're worded. And many of them contain flaws which I think will definitely make them unsuitable for modern LSATs. Even with this question, I don't think that D is really absolutely able to be inferred. Because there could possibly be another way of deterring war, that is much more efficient than military deterrence, that could work only when the other countries are unaware of its military power. So we must make the assumption that military deterrence do not have disadvantages.

    I hope that this answers your question, and to you and maybe others that read my comment, please let me know if there are any errors present in my explanations!

Sign In or Register to comment.